7
$\begingroup$

$\newcommand{\NS}{\mathrm{NS}}\mathcal{P}(\kappa)/I_{\NS}$ is the Boolean algebra of subsets of $\kappa$ under nonstationary symmetric difference, with $\mathbf{0} = [\emptyset] = I_{\NS}$, $\mathbf{1} = [\kappa] = I_{\NS}^*$ and $[X] \leq [Y]$ iff $X \setminus Y$ is nonstationary.

What is the maximum size of an ascending chain (well-ordered subset) in $\mathcal{P}(\kappa)/I_{\NS}$? Trivially it is at most $2^\kappa$, so $\kappa^+$ if $2^\kappa = \kappa^+$; is the $\kappa^+$ bound unconditional? Or can ascending chains of size $\kappa^{++}$ exist (from modest large cardinal strength)? This feels like something that would’ve been explored in the literature already but I can’t find any.

$\endgroup$
4
  • 2
    $\begingroup$ It is not obvious you can get an ascending chain of length $\kappa^+$. Whether this is possible is the question of the saturation of nonstationary ideal. $\endgroup$ Commented Sep 22 at 10:42
  • $\begingroup$ I was mostly thinking of large cardinals (e.g. weakly compact) $\kappa$, where an ascending chain of length $\kappa^+$ is easy. But, yeah, I suppose it’s interesting for $\kappa = \omega_1$ and such. $\endgroup$ Commented Sep 22 at 11:58
  • $\begingroup$ Anyway Gitik–Shelah showed ascending chains of length $\kappa^+$ exist (i.e., the nonstationary ideal is not saturated) for every regular uncountable $\kappa > \omega_1$. $\endgroup$ Commented Sep 22 at 13:06
  • 1
    $\begingroup$ Oh right, I just realized that my intuition of "ascending" / "descending" was muddled up. I had seen a characterization of saturation in terms of descending chains before and didn't realize that was the same as what I was asking for here. $\endgroup$ Commented Sep 22 at 16:29

1 Answer 1

10
$\begingroup$

It is consistent. Assume GCH. First, by iterating the $\kappa$-closed analogue of Hechler forcing, add an ascending chain in $\kappa^\kappa / \mathrm{bd}$ of length $\kappa^{++}$, and the areas under the graphs of these functions form an ascending $\subseteq^*$-chain in $\kappa \times \kappa$. Mapping these to subsets of $\kappa$ we get an ascending $\subseteq^*$-chain $\langle A_\alpha : \alpha < \kappa^{++} \rangle$ of subsets of $\kappa$.

Now add one Cohen subset of $\kappa$, getting a $\diamondsuit_\kappa$-sequence $\langle d_\alpha : \alpha<\kappa \rangle$. For each $\beta<\kappa^{++}$, let $B_\beta \subseteq \kappa$ be $\{ \alpha<\kappa : d_\alpha \subseteq A_\beta \}$. By the properties of $\diamondsuit$, each $B_\beta$ is stationary. It is also strictly increasing mod $\mathrm{NS}$ because if $\beta<\gamma$, then $\{ \alpha : d_\alpha = (A_\gamma \setminus A_\beta) \cap \alpha \}$ is stationary.

$\endgroup$
2
  • 1
    $\begingroup$ Wait, how is $B_\beta$ related to $\beta$? Is $d_\alpha \subseteq A_\alpha$ a typo of $d_\alpha \subseteq A_\beta$? $\endgroup$ Commented Sep 23 at 7:44
  • $\begingroup$ @JaydeSM Oh right, typo. Thanks. $\endgroup$ Commented Sep 23 at 11:20

You must log in to answer this question.

Start asking to get answers

Find the answer to your question by asking.

Ask question

Explore related questions

See similar questions with these tags.